22.2.25 (Flaw Questions Problem Set 24)

Accounts PlayableAccounts Playable Live Sage
edited November 2015 in Logical Reasoning 3107 karma
B is definitely a flaw in the argument, but can someone analyze my breakdown of D? Here is my breakdown:

50% of people in the survey believe that politician indicted----->politician resign.

35% believe that that politician resign----->politician convicted.

Therefore, more people think politician indicted----->politician resign than those that believe politician convicted---->politician resign.

What I am looking for: First, the conclusion is about "people" in general, but we are using a survey/poll. This introduces the possibility that the poll was biased/unrepresentative. Second, the conclusion makes a sufficiency/necessity conflation in the second comparative statement (about conviction). I didn't see this at first since, and I caught this flaw during BR.

Answer A: This is OK statistical/inductive reasoning. This would describe the flaw if the answer choice put the words "potentially biased" in front of sample.

Answer B: This is the correct answer since the 35% think resign--->convicted. But, the conclusion is about convicted--->resign. Pretty obvious answer choice if you read the last sentence carefully.

Answer C: What term is ambiguous? At best, the argument assumes that "politicians" and "elected officials" are the same thing, but that is an OK assumption.

Answer D: This is what I chose since I failed to see the sufficiency/necessity conflation originally. Would this be correct if the conclusion was correctly stated (if the comparative statement stated resign---->convicted)? Since the two responses convey different beliefs and since the argument is drawing a conclusion/comparison between them, is that a flaw? I am not so sure since the conclusion is about there being "more people" believing X than Y. Since both question were part of the same poll (and presumably same sample size) and since 50% is larger than 35% of that same sample size, would the argument have been valid (assuming there was not sample bias as well)?

Answer E: Why can't the premises all be true?
Sign In or Register to comment.